7. On Friday, Stock 1 dropped 3/4 point and Stock 2 dropped 5/8 point. Based on this information, which statement is true? Stock 2 dropped more Stock 1 dropped more The stocks dropped the same amount O Both stocks sold at the same price Clear selection

Answers

Answer 1

Let's divide each fraction

[tex]\begin{gathered} \frac{3}{4}=0.75 \\ \frac{5}{8}=0.625 \end{gathered}[/tex]

As you can observe, 3/4 is greater than 5/8.

Hence, Stock 1 dropped more.

Related Questions

Graph the function and then determine the asymptote of the function.

Answers

We need to graph the following function:

[tex]y=\log _{1/2}(x)-4[/tex]

We know that the domain of a logarithmic function can't be negative, then our domain is

[tex]x\in(0,\infty)[/tex]

We need to analyze this function at its extremes to find the asymptotes.

Let's calculate the limit of this function at x = 0 and at infinity.

[tex]\begin{gathered} \lim _{x\to0}\log _{1/2}(x)-4=\infty \\ \lim _{x\to\infty}\log _{1/2}(x)-4=-\infty \end{gathered}[/tex]

By definition, an asymptote of a curve is a line such that the distance between the curve and the line approaches zero as one or both of the x or y coordinates tends to infinity. From our limits, we have an vertical asymptote at x =0 and no horizontal asymptotes.

This logarithmic function, comes from infinity at x = 0, an decays to minus infinity as x grows. We have the following graph:

To prepare an aquarium for use, you can clean it with saltwater solution.The amount of salt varies directly with the volume of the water.The solution has 3 teaspoons of aquarium salt for every 2 gallons of water.

Answers

teaspoons of water = y

gallons of water = x

• a)

y = k x

Where k is the constant of proportionality.

Replace x,y by the values given and solve for k:

3= k 2

3/2 = k

k= 1.5

Equation:

y= 1.5x

• b) replace x=10 and solve for y

y= 1.5x

y= 1.5 (10)

y= 15

15 teaspoons of aquarium salt

• c) replace y= 39 and solve for x

y=1.5x

39 = 1.5 x

39/1.5= x

x = 26

26 gallons of water

The length of a rectangle is 1 inch shorter than twice the width (x).Which is the width (x) when the area (y) = 3321 square inches?

Answers

the width (x) of the rectangle = 41 inches

Explanation:

let the width = x

twice the width = 2x

The length of a rectangle is 1 inch shorter than twice the width (x) = 2x - 1

length = 2x -1

Area of rectangle = length × breadth

area (y) = 3321 square inches

y = x × 2x - 1 = x(2x - 1)

3321 = 2x² - x

2x² - x - 3321 = 0

We use factorisation to find x:

a = 2, b = -1, c = -3321

a × c = 2(-3321) = -6642

The factors which gives -1 when we sum together but gives -6642 when we multiply together are -82 and +81

2x² -82x + 81x - 3321 = 0

2x(x - 41) + 81(x - 41) = 0

(2x + 81) (x - 41) = 0

(2x + 81) = 0 or (x - 41) = 0

2x + 81 = 0

2x = -81

x = -81/2 inches

(x - 41) = 0

x - 41 = 0

x = 41 inches

Since we can't have a negative number as the width, the width (x) of the rectangle = 41 inches

Find each measure measurement indicated. Round your answers to the nearest tenth. Please show work. Answer number 1.

Answers

Let's redraw the given figure, to easily understood the problem:

The figure appears to be a triangle with the following given,

c = AB = 17 cm

a = BC = unknown

b = CA = 44 cm

θ = 125°

m∠B means it is the angle at vertex B of the triangle, it is also the only angle given in the figure.

Therefore, the measure of m∠B is 125°.

Jessica currently has $180 dollars in her bank account and will add an additional $15 each week. Nate has $120dollars in his account and will add $20 each week.A. After how many weeks will they have the same amount of money in their accounts?B. What is the amount, in dollars, that each person will have after this many weeks?

Answers

Answer:

(a)12 weeks

(b)$360

Explanation:

Part A

Let the number of weeks when they have the same amount of money in their accounts be x.

[tex]\begin{gathered} \text{Jessica's amount after x weeks }=180+15x \\ \text{Nate's amount after x weeks }=120+20x \end{gathered}[/tex]

If the amount of money is equal:

[tex]180+15x=120+20x[/tex]

Solve for x:

[tex]\begin{gathered} 180-120=20x-15x \\ 60=5x \\ \frac{60}{5}=\frac{5x}{5} \\ x=12 \end{gathered}[/tex]

Therefore, they will have the same amount of money after 12 weeks.

Part B

The amount that each person will have, (Using Jessica's Equation)

[tex]\begin{gathered} \text{Amount}=180+15x \\ =180+15(12) \\ =180+180 \\ =\$360 \end{gathered}[/tex]

The amount, in dollars, that each person will have after 12 weeks is $360.

A.Find the equation of the line of best fit round one decimal place, if needed choose the correct answer
B.the correlation coefficient is
C. The predicted number of cars sold in year 10 is

Answers

An equation for the line of best fit is equal to: D. y = 4.8x + 10.

The correlation coefficient, R² is equal to 0.988.

The predicted number of cars sold in year 10 is equal to 490 cars.

How to find an equation of the line of best fit for the data?

In order to determine a linear equation for the line of best fit (trend line) that models the data points contained in the table, we would have to use a graphing calculator (scatter plot).

In this scenario, the number of years would be plotted on the x-axis of the scatter plot while the number of cars sold would be plotted on the y-axis of the scatter plot.

On the Excel worksheet, you should right click on any data point on the scatter plot, select format trend line, and then tick the box to display an equation for the line of best fit (trend line) on the scatter plot.

From the scatter plot (see attachment) which models the relationship between the number of years and cars sold in the table, a linear equation for the line of best fit is given by:

y = 4.8x + 10

Next, we would determine the predicted number of cars sold in year 10:

y = 4.8x + 10

y = 4.8(10) + 10

y = 480 + 10

y = 490 cars.

Read more on scatter plot here: brainly.com/question/28605735

#SPJ1

Below is the graph of a trigonometric function. It intersects its midline at (-1.7, -10) and again at(5.1, -10). What is the period

Answers

you can get the Period graphically if you see the points A and B are the upper points of the graph. if you subtract them you get the period

in this case, let me see

Here they give you the points where the graph cross the midline, if you see, that is half of the period, so

Period= 2* (5,1-(-1,7))= 13,6

You have to subtract the points to know the distance between that points, and the graph is

when you subtract the points, you are getting the distance between (for example) the green point on my draw

And the period is the time it takes for one complete oscillation, after this, it repeats over and over

So between 3 of these green points we have a period

to know the value, we need to know the distance between them (in this case (5,1-(-1,7))= 6,8

each square have 1 period on it, and the value is 2 times the distance between the green points

Answer:

Period= 2* (5,1-(-1,7))= 13,6

This is a 1st-grade math problem 12 + _____ = 13 + 7

Answers

I'll say that the unknown number is "x", then our expression is

[tex]12+x=13+7[/tex]

And we want to find out the value of "x"

To solve it, we will first do the sum on the right side:

[tex]12+x=20[/tex]

Now, we want to have "x" alone on the left side, then, let's subtract 12 on both sides

[tex]\begin{gathered} 12-12+x=20-12 \\ \\ x=20-12 \\ \\ x=8 \end{gathered}[/tex]

Then the value of x is 8, let's test it:

[tex]\begin{gathered} 12+8=13+7 \\ \\ 20=20 \end{gathered}[/tex]

Correct! 12 + 8 is 20. Then, the unknown number is 8

Answer:

Step-by-step explanation:

12 + 1 = 13 + 7=20

wich of the following is the correct value of 0.22 0.4?

Answers

Answer:

The decimal place will be placed to the left by the total number of digits after the two numbers.

Explanation:

The multiplication we are asked to perform is

[tex]1.35\times4.2[/tex]

which has a total of 3 digits after the decimal point.

Now we know that

[tex]135\times42=5670[/tex]

therefore, to find 1.35 x 4.2 we just shift the decimal place in the above to the left by 3 units

therefore

[tex]1.35\times4.2=5.67[/tex]

which is our answer!

Here are the numbers of times 13 people ate out last month 7, 3, 4, 3, 6, 4, 7, 6, 5, 7, 3, 6,5Find the modes of this data set.If there is more than one mode, write them separated by commas.If there is no mode, tap on "No mode."Explanation

Answers

In statistics, the mode is the value that occurs most frequently in a data set. This goes in the form of a column when we find two modes, that is, two data that have the same maximum absolute frequency.

First, we are going to count how many times each number is repeated in the data set.

[tex]\begin{gathered} 7\to3\text{ times} \\ 3\to3\text{ times} \\ 4\to2\text{ times} \\ 6\to3\text{ times} \\ 5\to2\text{ times} \end{gathered}[/tex]

The highest frequency in the data set as we can see is 3. That is, the mode will be all the numbers that have that frequency in this case 7, 3 and 6

Which graph represents 7x+2y<8? four different graphs to chose from.

Answers

we have the inequality

7x+2y < 8

the solution for this inequality is the shaded area below the dashed line 7x+2y=8

so

the slope of the dashed line is negative

the intercepts of the dashed line are

y-intercept is (0,4)

the x-intercept is (8/7,0)

therefore

the answer is option B

Find the equation that results from completing the square into the following equation X squared -14 X +40 equals zero

Answers

Given:

The equation is

[tex]x^2-14x+40=0[/tex]

Required:

Find the equation that results from completing the square into the given equation.

Explanation:

The given equation is:

[tex]x^{2}-14x+40=0[/tex]

Subtract 40 on both sides.

[tex]\begin{gathered} x^2-14x+40-40=0-40 \\ x^2-14x=-40 \end{gathered}[/tex]

Add 49 on both sides.

[tex]\begin{gathered} x^2-14x+49=-40+49 \\ x^2-14x+49=9 \end{gathered}[/tex]

Use the following formula:

[tex]a^2-2ab+b^2=(a-b)^2[/tex][tex](x-7)^2=9[/tex]

Final answer:

The second option is the correct answer.

find the circumference to the nearest whole number the whole number is 14

Answers

Answer:

The circumference is 88 in

Explanation:

The circumference of a circle can be determined, using the formula:

[tex]C=2\pi r[/tex]

Where r is the radius of the circle.

Given a radius of 14 in, then

[tex]\begin{gathered} C=2(14)\pi \\ =28\pi \\ =28\times3.14 \\ =87.92 \\ \approx88in \end{gathered}[/tex]

Simplify or expand each expression and then Classify it by its degree and number of terms

Answers

2x^3 (7x^2 + 3x + 1) - 4x^4

multiply 2x^3 in

14x^5 + 6x^4 + 2x^3 - 4x^4

14x^5 + 2x^4 + 2x^3

3 terms, the degree is 5

terms are single expressions such as (2x^3)

the degree is the highest exponent

(6x - 2x^4 + 5x^6) + (7x^4 - 3x^6 + 9)

combine

6x - 2x^4 + 5x^6 + 7x^4 - 3x^6 + 9

simplify

2x^6 + 5x^4 + 6x + 9

4 terms, the degree is 6

(x^2 + 9)(x^2 - 9)

FOIL, (a + b)(c + d) -> ac + ad + bc + bd

x^2 * x^2 + x^2 * -9 + 9 * x^2 + 9 * -9

simplify

x^4 - 81

2 terms, the degree is 4

Find the surface area and volume of the figure .The surface area is _ft2.(Round to the nearest tenth as needed .)

Answers

Question:

Find the surface area and volume of the figure.

Solution:

1) The surface area:

This shape is composed of a cylinder and hemisphere. Now, we know that the surface area of the sphere is:

[tex]SA\text{ sphere = 4}\pi\text{ }r^2[/tex]

So that, the surface area of the hemisphere would be:

[tex]SA\text{ hemisphere = }2\pi r^2[/tex]

On the other hand, the area of the circle is:

[tex]A\text{= }\pi r^2[/tex]

thus, the surface area of the cylinder would be:

[tex]SA\text{ cylinder = }2\pi rh[/tex]

replacing the data given in the problem in the formulas of the surface area of the hemisphere, area of the circle, and surface area of the cylinder, we get:

[tex]SA\text{ hemisphere = }2\pi(9)^2\text{ = 162}\pi[/tex]

and

[tex]SA\text{ cylinder = }2\pi(9)(12)\text{ = }216\pi[/tex]

and

[tex]A\text{= }\pi(9)^2=\text{ 81}\pi[/tex]

then, we can conclude that the surface area of the given figure is:

[tex]SA\text{ = 162}\pi\text{ + 216}\pi+81\pi\text{ = 459}\pi\approx1441.9\text{ }\approx1442[/tex]

that is:

[tex]SA\text{ }\approx1441.9\text{ }\approx1442[/tex]

2) The volume

The volume of a cylinder is given by the following formula:

[tex]V_C=\pi r^2h[/tex]

and the volume of a hemisphere is :

[tex]V_H=\frac{1}{2}(\frac{4}{3}\pi r^3)\text{ = }\frac{2}{3}\pi r^3[/tex]

thus, the volume of the figure would be:

[tex]V=V_C+V_H=\text{ }\pi r^2h\text{+}\frac{2}{3}\pi r^3[/tex]

Then replacing the data given in the problem in the above formula we get:

[tex]V=\pi(9)^2(12)\text{+}\frac{2}{3}\pi(9)^3\text{ = 972}\pi\text{+486}\pi=\text{ 1458}\pi\approx4580.4\approx4580[/tex]

that is;

[tex]V\approx4580.4\approx4580[/tex]

micah runs a lemonade stand.He sells large cups of lemonade for$0.50 and small cups of lemonade for$0.35.Create an expression represents how much money he could earn

Answers

Answer:

The expression for the amount of money in dollars he could earn is;

[tex]0.50x+0.35y[/tex]

Explanation:

Let x represent the number of cups of large cups of lemonade he sell and

y represent the number of cups of the small cups of lemonade he sell.

Given that;

He sells large cups of lemonade for $0.50

and small cups of lemonade for $0.35.

The amount he will male by selling x cups of large cups of lemonade and y cups of small cups of lemonade is;

[tex]0.50x+0.35y[/tex]

The expression for the amount of money in dollars he could earn is;

[tex]0.50x+0.35y[/tex]

Answer:

the dude above me is wrong this is the right answer 0.85(l + s)

Step-by-step explanation:

Use add or sub formula to write as trig fun tho on

Answers

[tex]\frac{tan(43)-tan(18)}{1+tan(43)tan(18)}[/tex]

The formula to write this function is going to be:

[tex]tan(a-b)=\frac{tan(a)-tan(b)}{1+tan(a)*tan(b)}[/tex]

Substituing:

[tex]\frac{tan(43)-tan(18)}{1+tan(43)tan(18)}=tan(43-18)[/tex]

In this case a= 43 and b=18:

[tex]tan(43-18)=tan(25)=0.466\approx0.5[/tex]

The answer is: tan(25).

Destiny needs a new coat for the winter and she found one at Old Navy for $48.88. The sale tax is 8%. How much will she pay for the sales tax? How much will she pay all together? SHOW ALL OF YOUR WORK.

Answers

Hello!

First, let's consider the value of $48.88 as 100%. Then, we have to find how many correspond to 8% of it. We can calculate it using the rule of three:

[tex]\begin{gathered} \frac{48.88}{x}=\frac{100}{8} \\ \\ \text{ multiplying across} \\ x\cdot100=8\cdot48.88 \\ 100x=391.04 \\ x=\frac{391.04}{100} \\ x\cong3.91 \end{gathered}[/tex]

So, the sales tax is $3.91.

Now, we have to add the price of the coat with the sale tax:

$48.88 + $3.91 = $52.79

If she pays all together, the cost will be $52.79.

f(x) = -3x² + 6x + 1
find f(6)

Answers

Step-by-step explanation:

this is a college question ? and you don't know how to do this ?

come on, remember ! I don't know how you ever made it into college, if you don't understand this.

with f(6) we say x = 6, and now we simply have to put 6 into all the places of x and calculate.

f(6) = -3×6² + 6×6 + 1 = -3×36 + 36 + 1 = -2×36 + 1 =

= -72 + 1 = -71

The circumference of a circle is 37.68 meters, what is the radius?

Answers

[tex]\begin{gathered} \text{circumference}=37.68 \\ \text{radius}=\text{?} \\ \text{circumference of a circle =2}\pi r \\ 37.68=2\times3.14159r \\ r=\frac{37.68}{6.28318} \\ r=\text{ }5.99696332112 \\ r\approx6m \end{gathered}[/tex]

Michael says that 5 = 5 Is his answer correct? Explain. (1 O A. 00:00 Yes. Both expressions are equal to 625 СВ. 00:00 Yes Both expressions are equal to O C. 00:00 No. The first expression is equal to and the second expression is equal to 625 00:00 No. The first expression is equal to 625 and the second expression is equal to

Answers

Michael says that

[tex]5\cdot(\frac{1}{5^3})=5(5^3)[/tex]

We are asked whether he is correct or not?

Let us simplify the equa

Find the value of the logarithmic expression. log subscript 6 216log subscript 6 216=____?

Answers

Solution

For this case we have the following:

[tex]\log _6(216)=3[/tex]

The reason is because:

[tex]6^3=6\cdot6\cdot6=216[/tex]

Use the quadratic formula to solve for X.3x2 + 4x = 9Round your answer(s) to the nearest hundredth.Select all that apply.O x= 2.52x = 1.19x = -2.52x= -1.18O x = -20.00x = 20.00

Answers

[tex]3x^2+4x\text{ - 9 = 0}[/tex]

a = 3, b = 4, c = -9

[tex]x\text{ = }\frac{4\pm\text{ }\sqrt[]{4^2-4(3\times-9)}}{2\times3}[/tex][tex]x_1=2.52[/tex][tex]x_2\text{ = -1.1}8[/tex]

Drag each label to the correct location on the table. Each label can be used more than once, but not all labels will be used.after constant its 6x squared -x-1

Answers

For polynomial 1: simplified form

[tex]6x^2-x-1[/tex]

Name by Degree: Quadratic

Number of Terms: 3

Polynomial 2:

Simplified form is 3x+4

Name by Degree: Linear

Number of Terms: 2

Polynomial 3:

Simplified form: 2

Name by Degree: Number

Number of Terms: 1

[tex]3x+4[/tex]

Which expressions have the fewest significant Figures?A. 18.8 - 6.5B. 4350 - 2210C. 15.4 - 8.1D. 54.5 * 30.7

Answers

Answer

Option C is obviously the answer.

Explanation

It will be easy to answer this by providing the answers to the expressions.

Option A

18.8 - 6.5 = 12.3 (3 significant figures)

Option B

4350 - 2210 = 2140 (4 significant figures)

Option C

15.4 - 8.1 = 7.3 (2 significant figures)

Option D

54.5 * 30.7 = 1673.15 (6 significant figures)

Hope this Helps!!!

You have $28 to buy 7 goldfish for your new fish tank. Write and solve an inequality that represents the prices you can pay per fish.

Answers

If the price per each goldfish is p

The total cost of 7 goldfishes is 7p and it has to be less or equal to 28

The the inequality is:

7p <= 28

Solving for p:

7p <= 28

p <= 28/7 = 4

p <= 4

Answer:

You can pay up to $4 per each goldfish

Inequality: 7p <= 28

1. BC = 16 ft2. PQ = 22 cm139RS1с3. JK = 3 mm4. GH = 13 ydGK126845. YZ = 9 in6. EF = 28 mEF11542ZCG

Answers

Let's begin by listing out the given information:

The area of a sector is calculated using the formula:

1.

[tex]\begin{gathered} Area=\frac{\theta}{360^{\circ}}\times\pi r^2 \\ \theta=51^{\circ} \\ r=BC=16ft \\ \pi=3.14 \\ Area=\frac{51^{\circ}}{360^{\circ}}\times3.14\times16^2 \\ Area=113.88\approx114 \\ Area=114ft^2 \end{gathered}[/tex]

how much money should be deposited today in the account that are 7%, compounded and semi-annually so they will accumulate to 11,000 in 3 years

Answers

The rule of the compound interest is

[tex]A=P(1+\frac{r}{n})^{nt}[/tex]

A is the new amount

P is the initial amount

r is the rate in decimal

n is the number of the periods per year

t is the number of years

Since the rate is 7% compounded semi-annual, then

r = 7/100 = 0.07

n = 2

Since the amount after 3 years will be $11 000, then

A = 11 000

t = 3

Substitute them in the rule above to find P

[tex]\begin{gathered} 11000=p(1+\frac{0.07}{2})^{2(3)} \\ 11000=p(1.035)^6 \end{gathered}[/tex]

Divide both sides by (1.035)^6 to find P

[tex]\begin{gathered} \frac{11000}{(1.035)^6}=P \\ 8948.507087=P \end{gathered}[/tex]

Round it to the nearest cent (2 decimal places)

P = $8948.51

The amount invested was $8948.51

The total revenue from the sale of a poplar book is approximately by the rational function Where x us the number of years since publication and r(x) is the total revenue in millions of dollars. Use this function to complete parts a through dFind the total revenue at the of the first year ?

Answers

[tex]\begin{gathered} \text{tota revenue }at\text{ the end of 1st year } \\ R(1)=\frac{1200(1)^2}{1^{2+4}} \\ =\frac{1200}{5} \\ =240 \end{gathered}[/tex]

help ! brainliest !!!

Answers

answer:
a, the reflexive property
Other Questions
We want to factor the following expression: x^3 - 25 which pattern can we use to factor the expression? U and V are either constant integers or single variable expression. (a) How much power (in W) is dissipated in a short circuit of 225 V AC through a resistance of 0.280 ? W(b) What current (in A) flows? A 5. A ball is thrown from a platform. The equation h = -4.9t2 + 18t + 14 gives the ball's height, h, in meters in terms of time, t, in seconds. Part A: What was the initial velocity of the ball? Part B: From what height was the ball thrown? Part C: If we measure the height in feet, how would the function change? What would be the gravity coefficient? answe questions for just mercy chapter six unit 4: solving quadratic equationsHomework 9: quadratic equations applications Help please ! Arithmetic and Geometric Sequences (Context) I need help on this problem, its from my act prep guide How did monsoons facilitate maritime trade in the Indian Ocean?Question 10 options:predictable seasonal winds blew the invading Mongols back northpredictable seasonal winds made piracy very dangerouspredictable seasonal winds pushed trading ships in and out of the regionpredictable seasonal winds decreased the spread of the Black Death Find the value of x, if m Find the intersection if possibleExpress your answer in interval notation evaluate each inverse trig expression below. include a diagram of a unit circle justifying your solution. For the function f(x) = x2 + 2x - 15 solve the following.f(x) = 0 Aspirin is a common analgesic. If you want to produce 500. mg of aspirin (C9H8O4, m.w. = 180.16 g/mol) from the reaction of C7H6O3 (m.w. = 130.12 g/mol) and C4H6O3 (m.w. = 102.09 g/mol), what is the minimum amount of C4H6O3 that is needed?2C7H6O3(s) + C4H6O3(l) 2C9H8O4(s) + H2O(l) If trapezoid ABCD was dilated by a scale factor of 2\3 to form trapezoid A'B'C'D,what is the area of trapezoid ABCD?The area of trapezoid A'B'C'D is 12 units^2 evaluate each using the values given y+y-(y-x); use x = 1, and y = 4Options12154 Let g(y)= - y2 +9y + 10. Find g(-2). URGENT DUE TMR!!!!!Using your knowledge, experiences, and readings, write a carefully reasoned paragraph(s) in which you defend, challenge or qualify an assertion made by Golding on human nature. This should include an evaluation of Goldings arguments and specific claims and an assessment of the reasoning. Be sure to identity any false statements and fallacious reasoning and support your claim using relevant and sufficient evidence.I need 1 paragraph for each of theseA character that shows human beings are not fundamentally evilAn opposing claim paragraph Morgan wants to order at least $45 worth of merchandise, so she will get free shipping. If Morgan has picked out a key chain for $8 and a bag for $19, which inequality represents the amount of money, m, she needs to spend to get free shipping? You might need: Calculator h(r) = 72 +11r - 26 1) What are the zeros of the function? Write the smaller r first, and the larger r second. What is the percent by mass concentration of salt in a solution made by dissolving 3.5 g of salt in 75 kg of water?Please include equation used and variable you solved for.